Explore BrainMass

Explore BrainMass

    Optics

    BrainMass Solutions Available for Instant Download

    Geometrical Optics: Lenses, Prism, multiple-total reflection

    1. A Lighted candle (shown) 10.0 cm high is placed 40.0 cm in front of a converging lens with f1= 15 cm, which in turn is 70 cm in front of another converging lens of f2 = +20 cm. (a) Draw a "good" ray diagram "to scale" ( show focal lengths, etc.) for this problem. Estimate from your diagram the location and size of the fin

    Chapter 8: Activity-Based Costing: A Tool to Aid Decision Making

    Please refer to the attached word document. The following exercises need to be addressed: Exercise 8-10 Problem 8-14 Problem 8-18 Thanks. --- 356 Chapter 8 Activity-Based Costing: A Tool to Aid Decision Making EXERCISE 8-10 Comprehensive Activity-Based Costing Exercise Silicon Optics has supplied the fol

    Ray Optics: Combination of lens and mirror

    An object is placed 12.0 cm in front of a converging lens of focal length 5.00 cm. A concave mirror with radius of curvature 20.0 cm is located 30.0 cm in back of the lens as shown. (refer to attached picture) 1.) At what distance from the lens does the first image (from the lens) form? 2.) What is the magnification of the

    Optics/Prism problem

    A light beam enters a 30 - 60 - 90 degree plastic prism of index of refraction 1.45 with an angle of 32.8o with the face as shown. The prism is in air. (refer to attached picture) 1.) Sketch the path of the beam as it passes through the prism. 2.) Find the angle the beam makes with the diagonal surface after it exits the p

    Ray Optics: Combination of lenses

    A diverging lens (f = -10.5 cm) is located 20.0 cm to the left of a converging lens (f = 32.0 cm). A 3.00 cm tall object stands to the left of the diverging lens, exactly at its focal point. (a) Determine the distance of the final image relative to the converging lens. (b) What is the height of the final image (including

    Telescope Optics

    A severely near-sighted sailor on a ship is asked to be on the lookout for icebergs on the horizon. To do so, he is given a Newtonian Telescope with f(objective) = 30cm and f(eyepiece)= 5 cm (see diagram in attached file). a. If the sailor's relaxed eye focuses objects at a distance of 1m (his far-point distance), how far apart

    Laser beam enters a prism

    (See attached file for full problem description with diagram) --- 8. A laser beam enters one of the sloping faces of the equilateral glass prism (n = 1.41) in the figure below and refracts through the prism. Within the prism the light travels horizontally. What is the angle between the direction of the incident ray and the

    'OPTICS'

    4.15 Light of wavelength 600 nm in vacuum enters a block of glass where n_g=1.5. Compute its wavelength in the glass. What color would it appear to someone embedded in the glass? See attached 4.19 A laserbeam having a diameter D in air strikes a piece of glass (n_g) at an angle theta_i. What is the diameter of the beam in

    Optics of thick glass mirror

    Images in a very thick glass mirror may by slightly colored by dispersion. Draw two vertical lines, separated by about 2 cm, to represent the front (glass) surface and the rear (silver) surface of the mirror. About 2 cm in from of the mirror, draw the object-a white point source. (a) Carefully draw red and blue rays to represent

    Optics of Lenses

    You are holding a lens in your hand. How can you tell if it is: (a) positive (converging) (b) negative (diverging), or (c) neither (a) nor (b) above... draw pictures to show...

    Optics/light

    When you look at a white light source through a rainbow peephole, why is ONLY the central spot white?

    Analyzing Optics and Waves

    Question: 5-4 Sketch Diagram attached... a) What is the critical angle theta of the interface? b) What is the acceptable angle 'a'. Answer to the problem. (See attached file for full problem description).

    Optics and Waves Question

    Question: Radio waves of frequency 900 kHz are broadcast in step from two stations 15 kilometres apart. a) What is the wavelength of the signals? (c = 3 multiplied by 10`8 ms-1) b) What is the path difference between the two waves received at a point P 6 kilometres from one station along a line directly towards the oth

    Calculate the Path of Light

    Question: Light from a torch held under water by a diver hits the surface at angle of incidence a) 40 degrees b) 55 degrees In each case calculate the path of the light. (Refractive index of water = 1.33) I would like the workings and answer to this problem please.

    Optics

    Objective: To examine how images can be formed from the light of distant sources. Equipment: Example lens (magnifying, reading glasses, or eyeglasses for a farsighted person), a small hand mirror, blank paper; straight edge (ruler), protractor. Description: Quantitative interpretation of astronomical images requires a k

    Light and Optics

    1. Light is incident at angle A from a media with n = 1.5 to one with n = 2.0 as shown in the figure. What is the minimum value for A so that there is total internal reflection at point P, which is at the interface between the n = 2 and n = 1 materials? ? A

    Physical Optics/resolution

    Hi. Can someone please show me how to do the following problem? Thank you. "When reading the printout from a laser printer, you are actually looking at an array of tiny dots. If the pupil of your eye is 2.1 mm in diameter when reading a page held 33 cm from your eye, what is the minimum separation of adjacent dots that can be

    Physical Optics/constructive and destructive reflection

    Hi. Can someone please show me how to do the following problem? Thank you. White light is incident normally on a thin soap film (n = 1.33). (a) What are the two minimum thicknesses that will constructively reflect visible light with = 676 nm? smallest thickness (in nm) second smallest thickness (in nm) (b) What

    Physical Optics and Interference

    Moe, Larry, and Curly sit in a line with a spacing of 1.00 m. Larry is d1 = 3.0 m in front of a pair of stereo speakers d2 = 0.800 m apart, as shown in Figure 28-33. The speakers produce a single-frequency tone, vibrating in phase with each other. What are the two lowest frequencies that allow Larry to hear a loud tone while Moe

    Problems with Optics and Lenses

    Hi. Can someone please show me how to do the following problem? Thank you. "A magnifying glass is a single convex lens with a focal length of f = +11.0 cm. (Assume the person using the magnifying glass has a near point of 25 cm and the magnifying glass is directly in front of the person's eyes.) (a) What is the angular mag

    Optics and Lenses

    Hi. can someone please show me how to do the following problem? Thank you. "With unaided vision, a librarian can focus only on objects that lie at distances between 6.0 m and 0.37 m. Find the refractive power needed for each part of the bifocal eyeglass lenses (1. for the converging lens and 2. for the diverging lens) that wi

    Optics and Diffraction Problem Set

    1) The drawing shows a crystalline slab (n = 1.402) with a rectangular cross section. A ray of light strikes the slab at an incident angle of theta_1 = 44 degree, enters the slab, and travels to point P. This slab is surrounded by a fluid with a refractive index n. What is the maximum value of n such that total internal reflecti

    Optics

    An electron is placed in a constant electric field of magnitude 800 N/C. What is the acceleration of the electron and the electromagnetic power radiated by the electron? I'm not sure how to set it up. Can you help?

    Optics - Radiation Intensity

    A laser can be used to suspend beads in the earth's gravitational field. A bead with mass 5 * 10^-6 g and density of .400 g/cm^3 is to be suspended. Determine the radiation intensity needed to support the bead. I'm not sure how to set it up. Help please?

    Wave Optics: Interference and Diffraction

    1.A circular diffraction pattern is formed on a faraway screen. (a) By what factor will the width of the central maximum change if the wavelength is doubled? (a) 4 (b) 2 (c) 1 (no change) (d) 1/2 (e) 1/4 (b) By what factor will the area of the central maximum change if the slit